LSAT and Law School Admissions Forum

Get expert LSAT preparation and law school admissions advice from PowerScore Test Preparation.

 Administrator
PowerScore Staff
  • PowerScore Staff
  • Posts: 8916
  • Joined: Feb 02, 2011
|
#27447
Complete Question Explanation

Must Be True—SN. The correct answer choice is (C)

This stimulus says that if a tax raises revenue and burdens just/all people targeted by the tax, then it will be effective (RR + Burden :arrow: Eff). It then states that if a tax does not raise revenue and costs money to enforce it will not be effective (RR + $$ :arrow: Eff). The correct answer choice could come from either of these two ideas.

Answer choice (A): We can only know the tax is effective if it burdens all those taxed, so this is not necessarily true.

Answer choice (B): The tax will be ineffective if it does not raise revenue, so if it raises a modest amount we cannot know that it will be ineffective.

Answer choice (C): This is the correct answer choice. This answer follows directly from the second sentence (RR + $$ :arrow: Eff).

Answer choice (D): Without any knowledge of revenue we cannot know if the tax will be effective or not.

Answer choice (E): We can only know the tax is ineffective if it does cost a significant amount of money to enforce.
 Toby
  • Posts: 33
  • Joined: Jun 05, 2017
|
#37705
Hello!

I have a question about trying to link the two SN statements together in this stimulus. I attempted to link them together by using the contrapositve of the first statement. Here is what my diagram looked like

NOT RR + $$ :arrow: NOT eff :arrow: NOT RR or NOT burden

I realize that this type of chain isn't necessary for finding the correct answer to this question, but I would just like to check whether my logic is correct in making this chain. Is it possible to use a contrapositive in order to link two statements? Thanks for the help!

Toby
 Eric Ockert
PowerScore Staff
  • PowerScore Staff
  • Posts: 164
  • Joined: Sep 28, 2011
|
#37719
Toby

Your logic is correct there. You can use the contrapositive to connect the two. And good catch on the "or" connection on the back end of that chain. That is the proper contrapositive, and the proper chain.

But you're right...this chain doesn't do you much good for this question. :)
 Toby
  • Posts: 33
  • Joined: Jun 05, 2017
|
#37738
Thanks for checking my work for me, Eric!

Get the most out of your LSAT Prep Plus subscription.

Analyze and track your performance with our Testing and Analytics Package.